Quantcast
  • Register
PhysicsOverflow is a next-generation academic platform for physicists and astronomers, including a community peer review system and a postgraduate-level discussion forum analogous to MathOverflow.

Welcome to PhysicsOverflow! PhysicsOverflow is an open platform for community peer review and graduate-level Physics discussion.

Please help promote PhysicsOverflow ads elsewhere if you like it.

News

PO is now at the Physics Department of Bielefeld University!

New printer friendly PO pages!

Migration to Bielefeld University was successful!

Please vote for this year's PhysicsOverflow ads!

Please do help out in categorising submissions. Submit a paper to PhysicsOverflow!

... see more

Tools for paper authors

Submit paper
Claim Paper Authorship

Tools for SE users

Search User
Reclaim SE Account
Request Account Merger
Nativise imported posts
Claim post (deleted users)
Import SE post

Users whose questions have been imported from Physics Stack Exchange, Theoretical Physics Stack Exchange, or any other Stack Exchange site are kindly requested to reclaim their account and not to register as a new user.

Public \(\beta\) tools

Report a bug with a feature
Request a new functionality
404 page design
Send feedback

Attributions

(propose a free ad)

Site Statistics

205 submissions , 163 unreviewed
5,047 questions , 2,200 unanswered
5,345 answers , 22,709 comments
1,470 users with positive rep
816 active unimported users
More ...

  What is a general definition of the spin of a particle?

+ 7 like - 0 dislike
1735 views

In quantum field theory, one defines a particle as a unitary irreducible representations of the Poincaré group. The study of these representations allows to define the mass and the spin of the particle. However, the spin is not defined the same way for massive particles (where the eigenvalue of the Pauli-Lubanski vector squared are $-m^2 s(s+1)$ where $$s = -S, -S +1, \cdots, S,$$ and $S$ is the spin of the particle (and $m$ the mass)) and massless particles (where the helicity has eigenvalues $\pm \lambda$ and $S=\left|\lambda\right|$ - not to mention continuous spin representations).

With this definition, the "spin" $S$ that appears in both cases doesn't seem to be exactly the same thing. The eigenvalue that labels the irreducible representations are not from the same operator in the massive and the massless case ... however, it's tempting (for me) to see the maximum value of these eigenvalues as the same physical quantity if both cases.

So I wonder if there is a more general definition that would embrace both massive and massless particles (even less practicable)?

This post imported from StackExchange Physics at 2016-02-15 17:16 (UTC), posted by SE-user Georg Sievelson
asked Mar 16, 2012 in Theoretical Physics by Georg Sievelson (35 points) [ no revision ]
retagged Feb 15, 2016
You can think of spin as residual angular momentum in the rest frame.

This post imported from StackExchange Physics at 2016-02-15 17:16 (UTC), posted by SE-user Antillar Maximus
@AntillarMaximus Well it does not work for a photon (or any massless particle), as it has no rest frame.

This post imported from StackExchange Physics at 2016-02-15 17:16 (UTC), posted by SE-user Georg Sievelson
Related: physics.stackexchange.com/q/1/2451

This post imported from StackExchange Physics at 2016-02-15 17:16 (UTC), posted by SE-user Qmechanic
Ultimately the only reason we think of the angular momentum of a gyroscope, the spin of an electron, and the spin of a photon as related to one another is that they couple to one another. The fact that they couple means that only their sum is conserved; they're not conserved individually.

This post imported from StackExchange Physics at 2016-02-15 17:16 (UTC), posted by SE-user Ben Crowell

You can think of spin as residual angular momentum in the rest frame.

I just wanted to note that the rest frame means $\mathbf{P}=0$, not $\mathbf{R}=0$, so the particle can be found everywhere.

@VladimirKalitvianski: Indeed, in the rest frame, the particle is ''here'', wherever this is. The relative positions of other particles commute with $\P$, hence are well determined in the tangent space of the rest frame.

1 Answer

+ 6 like - 0 dislike

In general, quantum numbers are labels of irreducible representations of the relevant symmetry group, not primarily eigenvalues of an otherwise simply defined operator.

But for every label that has a meaningful numerical value in every irreducible representation, one can define a Hermitian operator having it as an eigenvalue, simply by defining it as the sum of the projections to the irreducible subspaces multiplied by the label of this representation. It is not clear whether such an operator has any practical use.

This also holds for the spin. However, one can define the spin in a representation independent way, though not via eigenvalues.

The spin of an irreducible positive energy representation of the Poincare group is $s=(n-1)/2$, where $s$ is the smallest integer such that the representation occurs as part of the Foldy representation in $L^2(R^3,C^n)$ with inner product defined by
$~~~\langle \phi|\psi \rangle:= \displaystyle \int \frac{dp}{\sqrt{p^2+m^2}} \phi(p)^*\psi(p)$.
The Poincare algebra is generated by $p_0,p,J,K$ and acts on this space as follows. $p$ is multiplication by $p$,
$~~~p_0 := \sqrt{m^2+p^2}$,
$~~~J := q \times p + S$,
$~~~K := \frac{1}{2}(p_0 q + q p_0) + \displaystyle\frac{p \times S}{m+p_0}$,
with the position operator $q := i \hbar \partial_p$ and the spin vector $S$ in a unitary irreducible representation of $so(3)$ on the vector space $C^n$ of complex vectors of length $n$, with the same commutation relations as the angular momentum vector.

The Poincare algebra is generated by $p_0,p,J,K$ and acts on this space irreducibly if $m>0$ (thus givning the spin $s$ representation), while it is reducible for $m=0$. Indeed, in the massless case, the helicity
$~~~\lambda := \displaystyle\frac{p\cdot S}{p_0}$,
is central in the universal envelope of the Lie algebra, and the possible eigenvalues of the helicity are $s,s-1,...,-s$, where $s=(n-1)/2$. Therefore, the eigenspaces of the helicity operator carry by restriction unitary representations of the Poincare algebra (of spin $s,s-1,...,0$), which are easily seen to be irreducible.

The Foldy representation also exhibits the massless limit of the massive representations.

Edit: In the massless limit, the formerly irreducible representation becomes reducible. In a gauge theory, the form of the interaction (multiplication by a conserved current) ensures that only the irreducible representation with the highest helicity couples to the other degrees of freedom, so that the lower helicity parts have no influence on the dynamics, are therefore unobservable, and are therefore ignored.

This post imported from StackExchange Physics at 2016-02-15 17:16 (UTC), posted by SE-user Arnold Neumaier
answered Mar 16, 2012 by Arnold Neumaier (15,787 points) [ no revision ]
Well, thanks. Do you have any reference about that ? I found a 1956 article from L. Foldy, but it only deals with massive particles. Aside from that, I understand it is probably a too much blurry question, but do you have some physical explanation of why the quantity that happens to be the spin of the particle in all cases is the same ?

This post imported from StackExchange Physics at 2016-02-15 17:16 (UTC), posted by SE-user Georg Sievelson
I don't think he ever considered the massless case, though it is clear that he gets a representation if m=0. I figured out the details for myself. So if you need a reference, cite the section ''Particle positions and the position operator'' of Chapter B1: The Poincare group of my <A HREF="mat.univie.ac.at/~neum/physfaq/physics-faq.html"&gt; theoretical physics FAQ</A>

This post imported from StackExchange Physics at 2016-02-15 17:17 (UTC), posted by SE-user Arnold Neumaier
''the same'' makes no sense. It is usually called by the same name, as it contributes to the total angular momentum. But sometimes it is said that a photon has no spin byt only helicity....

This post imported from StackExchange Physics at 2016-02-15 17:17 (UTC), posted by SE-user Arnold Neumaier
Well, I try to understand to what extent they are similar. But the suppression of the 0-helicity mode in the massless limit somehow fulfils my need of a cloudy physical explanation. Thanks.

This post imported from StackExchange Physics at 2016-02-15 17:17 (UTC), posted by SE-user Georg Sievelson
I added to my answer another comment on the massless limit, to clarify why the suppression occurs.

This post imported from StackExchange Physics at 2016-02-15 17:17 (UTC), posted by SE-user Arnold Neumaier

Your answer

Please use answers only to (at least partly) answer questions. To comment, discuss, or ask for clarification, leave a comment instead.
To mask links under text, please type your text, highlight it, and click the "link" button. You can then enter your link URL.
Please consult the FAQ for as to how to format your post.
This is the answer box; if you want to write a comment instead, please use the 'add comment' button.
Live preview (may slow down editor)   Preview
Your name to display (optional):
Privacy: Your email address will only be used for sending these notifications.
Anti-spam verification:
If you are a human please identify the position of the character covered by the symbol $\varnothing$ in the following word:
p$\hbar$ysicsO$\varnothing$erflow
Then drag the red bullet below over the corresponding character of our banner. When you drop it there, the bullet changes to green (on slow internet connections after a few seconds).
Please complete the anti-spam verification




user contributions licensed under cc by-sa 3.0 with attribution required

Your rights
...